A cork dropped into a water filled beaker floats with volume V1 representing the portion of cork above water. When it is dropped in a beaker containing corn syrup, it floats with its volume V2 (again, the portion of cork above syrup). How do these volumes compare?A. V1 = V2.B. V1 > V2.C. V1 < V2.D. V1 ≥ V2.

Answers

Answer 1

Answer:

C. V1 < V2

Explanation:

The computation is shown below:

As we know that

Byoyancy force represent the displaced water weight

[tex]B = V_{in},_{w}P_{w}g[/tex]

[tex]V_{in},_{w}[/tex] denotes cork volume that inside the water

[tex]P_{w}[/tex] denotes water density

And, byoyancy force represent the displaced weight of corn syrup

[tex]B = V_{in},_{syr}P_{syr}g[/tex]

[tex]V_{in},_{syr}[/tex] denotes the cork volume that inside the water

[tex]P_{syr}[/tex] denotes syrup density

Now

[tex]P_{syr}>P_{w}\\\\V_{in},_{syr}<V_{in},_{wat}\\\\V_2>V_1 or V_1 <V_2[/tex]

Hence, the option c is correct


Related Questions

A 20 cm radius ball is uniformly charged to 70 nC.
(a) What is the ball's charge density?
(b) How much charge is enclosed by spheres of radii 5, 10 and 20 cm?
(c) What is the electric field strength at points 5, 10 and 20 cm from the center?

Answers

Answer:

Explanation:

A)

Density= charge/total volume .......eqn(1)

But volume= 4/3πr^3

r= radius= 20 cm= 0.20m

If we substitute into the volume equation, we have

volume= 4/3 * 3.142 *( 0.20)^3

= 0.0335 m^3

The volume= 0.0335 m^3

Charge=71 nC= 71×10^-9

If we substitute into eqn(1) we have

Density= (71 *10^-9C )/0.0335

= 2.11µc/m^3

B) charge enclose= Density × volume

spheres of radii are

5cm

10 cm

20 cm

Volume for 5cm

V= 4/3 * 3.142 *( 0.05)^3 = 0.0005237 m^3

charge enclose=2.11µc/m^3×0.0005237

charge enclose= 2.110 nC

Volume for 10cm

V= 4/3 * 3.142 *( 0.10)^3 = 0.004189 m^3

charge enclose= 2.11µc/m^3 ×0.004189

=8.9 nC

Volume for 20cm

V= 4/3 * 3.142 *( 0.20)^3 = 0.0335 m^3

charge enclose= 71nC

pers
2. (a) Calculate the virtual depth of a black dot at the
bottom of a cubic block made of transparent glass with each
side 4 cm, while the refractive index of glass is 1.6.​

Answers

Answer:

2.5 cm

Explanation:

Using the relation :

Refractive index = Real Depth / Apparent depth

Refractive index = 1.6

Real depth = 4cm

Virtual depth = apparent depth = x

1.6 = 4cm / x

1.6x = 4

x = 4 / 1.6

x = 2.5

Hence, virtual depth = 2.5cm

Light rays travel from one medium into another and refract away from the boundary. What changes about the light to cause this refraction?

A. Its speed increases.
B. Its frequency increases.
C. Its frequency decreases.
D. Its speed decreases.​

Answers

Answer:

a

Explanation:

I think its a its speed increases.....

coefficient of static friction formula

Answers

In coefficient of static friction, force is directly proportional to normal reaction, that means:

F=M N (consider M as coefficient of static friction)

now,

M=F/N

If you'll see the unit, then unit of F is Newton and unit of N is also Newton.

Therefore,

M= newton/newton

Therefore, Coefficient of static friction is unitless.

What body system,
respiratory system
Circulatory system
Digestive system
Nervous system


Affects ulcer disease and heartburn.


Please do this quick it is due please. I will make brainlest and give out extra points


Affects ulcer disease and heartburn.


Please do this quick it is due please. I will make brainlest and give out extra points!

Answers

Answer:

Digestive system

Explanation:

ulcer affect anywhere in the digestive system

Digestive system.

Since the acids in your food break down with the chemicals in your stomach, it can give you heartburn and also, ulcer disease happens in your stomach so the only correct answer would be Digestive System. I would like to say that the person with the profile name BigPapa who commented on my answer deserves a lot of credit, and thanks if you see this.

-R3TR0 Z3R0

If the mass of the object doubles then the acceleration is when the force is kept the same

Answers

Answer:

Halved

Explanation:

F=ma

Let case 1 (original) be:

[tex]F_{1}=m_{1} a_{1} \\[/tex]

Case 2 (new) be:

[tex]F_{2}=m_{2} a_{2}[/tex]

Mass is double:

[tex]m_{2}= 2m_{1}[/tex]

Force kept the same:

[tex]F_{1} =F_{2}[/tex]

Combine the equation and gives:

[tex]\frac{F_{1} }{F_{2}} =\frac{m_{1} a_{1} }{m_{2}a_{2} }\\\frac{F_{1} }{F_{1}} =\frac{m_{1} a_{1} }{2m_{1}a_{2} }\\1=\frac{a_{1} }{2a_{2} }\\a_{2}=\frac{1}{2} a_{1}[/tex]

Acceleration is halved

Which graph best represents the greatest amount of work

Answers

Where are the graphs please they are not showing I really wanna help

A ball is thrown vertically downward from the top of a 37.4-m-tall building. The ball passes the top of a window that is 15.4 m above the ground 2.00 s after being thrown. What is the speed of the ball as it passes the top of the window?

Answers

Answer:

v= 20.8 m/s

Explanation:

Assuming no other forces acting on the ball, from the instant that is thrown vertically downward, it's only accelerated by gravity, in this same direction, with a constant value of -9.8 m/s2  (assuming the ground level as the zero reference level and the upward direction as positive).In order to find the final speed 2.00 s after being thrown, we can apply the definition of acceleration, rearranging terms, as follows:

       [tex]v_{f} = v_{o} + a*t = v_{o} + g*t (1)[/tex]

We have the value of t, but since the ball was thrown, this means that it had an initial non-zero velocity v₀.Due to we know the value of the vertical displacement also, we can use the following kinematic equation in order to find the initial velocity v₀:

        [tex]\Delta y = v_{o} *t + \frac{1}{2} * a* t^{2} (2)[/tex]

where Δy = yf - y₀ = 15.4 m - 37.4 m = -22 m (3)Replacing by the values of Δy, a and t, we can solve for v₀ as follows:

       [tex]v_{o} = \frac{(\Delta y- \frac{1}{2} *a*t^{2})}{t} = \frac{-22m+19.6m}{2.00s} = -1.2 m/s (4)[/tex]

Replacing (4) , and the values of g and t in (1) we can find the value that we are looking for, vf:

       [tex]v_{f} = v_{o} + g*t = -1.2 m/s - (9.8m/s2*2.00s) = -20.8 m/s (5)[/tex]

Therefore, the speed of the ball (the magnitude of the velocity) as it passes the top of the window is 20.8 m/s.

HELP ASAP WILL GIVE BRAINLIEST TO WHOEVER ANSWERS FIRST!!!
A car with a mass of 1,200 kg accelerates at a rate of 3.0 m/s^2 forward. What is the force acting on the car?

Answers

Answer:

the force acting on the car is 3600 N

Explanation:

The computation of the force acting on the car is shown below:

As we know that

Force = mass × acceleration

= 1200 kg × 3.0 ms/^2

= 3600 N

hence, the force acting on the car is 3600 N

Balanced forces acting on an object keeps it at ____or moving at _____in straight line.

Fill in the ___ spaces

Answers

Answer:

Please see below as the answer is self explanatory.

Explanation:

According to Newton's 2nd law,  a net force acting on an object of mass m, causes the object to be accelerated.If the forces acting on the object are balanced, which means that the net force on the object is zero, just applying the same law, we find that the object is not accelerated.According to Newton's First law, an object that is not accelerated is at rest, or moves along a straight line at constant speed.So, if there are balanced forces acting on the object, if the object is at rest, will keep at rest, and if it is moving, it will keep moving at constant speed along a straight line.

What force is needed to give a 4800.0 kg truck an acceleration of 6.2 m/s2 over a level road? ​

Answers

Answer:

the force needed to give the truck the acceleration is 29,760 N.

Explanation:

Given;

mass of truck, m = 4800 kg

acceleration of the truck, a = 6.2 m/s²

The force needed to give the truck the acceleration is calculated as;

F = ma

F = 4800 x 6.2

F = 29,760 N

Therefore, the force needed to give the truck the acceleration is 29,760 N.

what was Thomas Edison first major invented? ​

Answers

Answer:

Thomas Edisons most famous invention was the phonograph

Thomas Edison announces his invention of the phonograph, a way to record and play back sound. Edison stumbled on one of his great inventions—the phonograph—while working on a way to record telephone communication at his laboratory in Menlo Park, New Jersey.

Explanation:

Hope I helped

An iron nail becomes a permanent magnet if it is

Answers

if you stroke it an iron nail with a bar magnet the nail will become a permanent or long lasting magnet.

Hope it's perfect for you.

unpolarized light of intensity Io is incident on an ideal linear polariser (no absorption) . what is the transmitted intensity?​

Answers

Answer:

A Polarizing sheet transmits only the component of light polarized along a particular direction and absorbs the component perpendicular to that direction.

Consider a light beam in the z direction incident on a Polaroid which has its transmission axis in the y direction. On the average, half of the incident light has its polarization axis in the y direction and half in the x direction. Thus half the intensity is transmitted,and the transmitted light is linearly polarized in the y direction.



Name the nutrients required for the
body.​

Answers

Answer:

1- water

2- fat

3- carbohydrates

4- vitamins

5- minerals

You and a friend each hold a lump of wet clay. Each lump has a mass of 25 grams. You each toss your lump of clay into the air, where the lumps collide and stick together. Just before the impact, the velocity of one lump was < 4, 4, -3 > m/s, and the velocity of the other lump was < -2, 0, -7 > m/s.
What was the the total momentum of the lumps just before the impact?
p(total) = ____kg·m/s.
What is the momentum of the stuck-together lump just after the collision?
p = ____kg·m/s.
What is the velocity of the stuck-together lump just after the collision?
v_f = ____m/s.

Answers

Answer:

a) p(total) = <0.05, 0.1, 0.1 > kg m/s

b) p = <0.05, 0.1, 0.1 > kg.m/s

c) v_f = < 1, 2, 2 > m/s

Explanation:

a.)

Mass of each lump = 25 g = 0.025 kg

Velocity of lump 1 = < -2, 0, -7 > m/s

Momentum of lump 1 = Mass×Velocity

                                   = 0.025×< -2, 0, -7 >

                                   = < -0.05, 0, 0.175> kg m/s

Velocity of lump 2 = < 4, 4, -3 > m/s

Momentum of lump 2 = Mass×Velocity

                                    = 0.025×< 4, 4, -3 >

                                    = < 0.1, 0.1, -0.075> kg m/s

Total momentum before impact  =  < -0.05,  0,  0.175 > + < 0.1, 0.1, -0.075>

                                                      = < 0.05, 0.1, 0.1 > kg m/s

⇒p(total) = <0.05, 0.1, 0.1 > kg m/s

b)

As we know that,

By the law of conservation of linear momentum,

The total momentum will be the same before and after the collision.

⇒Momentum of the stuck together  after the collision = Total momentum of the lumps just before impact.

⇒ p = <0.05, 0.1, 0.1 > kg m/s

c)

Let the final velocity =  v_f

Total mass = 0.025 + 0.025 = 0.05 kg

As

Momentum = mass ×velocity

⇒ <0.05, 0.1, 0.1 > = 0.05 ×v_f

⇒ v_f = <0.05, 0.1, 0.1 > / 0.05

          = < 1, 2, 2 > m/s

⇒v_f = < 1, 2, 2 > m/s

If the magnitude of vector A⃗ is less than the magnitude of vectorB⃗ , then the x component of A⃗ is less than the x component ofB⃗ . If the magnitude of vector is less than the magnitude of vector, then the component of is less than the component of.



a. True


b. False

Answers

False ?
I’m not totally sure but I think false

An ideal heat engine operates between 778 K and 475 K. 267 J of waste heat is exhausted. What is the input heat?

Answers

Answer:

Explanation:

Suppose that the turbines of a coal-fired plant are driven by hot gases at a temperature of 886 K. the temperature of the exhaust area is only 305 K,  the efficiency of this heat engine

A uranium ion and an iron ion are separated by a distance of =61.10 nm. The uranium atom is singly ionized; the iron atom is doubly ionized. Calculate the distance from the uranium atom at which an electron will be in equilibrium. Ignore the gravitational attraction between the particles. = nm An electron sits between a singly ionized uranium ion and a doubly ionized iron ion. The distance from the uranium ion to the electron is designated lowercase r, and the distance between the two ions is designated uppercase R. What is the magnitude of the force on the electron from the uranium ion? magnitude of the force: N

Answers

Answer:

Explanation:

Charge on uranium ion = charge of a single electron

= 1.6 x 10⁻¹⁹ C

charge on doubly ionised iron atom = charge of 2 electron

= 2 x 1.6 x 10⁻¹⁹ C = 3.2 x 10⁻¹⁹ C

Let the required distance from uranium ion be d .

force on electron at distance d from uranium ion

= 9 x 10⁹ x 1.6 x 10⁻¹⁹ / r²

force on electron at distance 61.10 x 10⁻⁹ - r from iron  ion

= 9 x 10⁹ x 3.2 x 10⁻¹⁹ / (61.10 x 10⁻⁹ - r )²

For equilibrium ,

9 x 10⁹ x 1.6 x 10⁻¹⁹ / r² = 9 x 10⁹ x 3.2 x 10⁻¹⁹ / (61.10 x 10⁻⁹ - r )²

2 d² = (61.10 x 10⁻⁹ - r )²

1.414 r = 61.10 x 10⁻⁹ - r

2.414 r = 61.10 x 10⁻⁹

r = 25.31 nm .

(a)  The distance from the uranium atom at which an electron will be in equilibrium is [tex]2.04 \times 10^{-8} \ m[/tex]

(b) The magnitude of the force on the electron from the uranium ion is [tex]3.46 \times 10^ 6 \ N[/tex]

The given parameters:

distance between the iron and the uranium, d = 61.1 nmcharge of uranium ion, q₁ = 1.6 x 10⁻¹⁹ Ccharge of doubly ionized atom, q₂ = 2q₁ = 3.2 x 10⁻¹⁹ C

The force on the electron due to uranium ion at distance r is calculated as follows;

[tex]F _1 = \frac{Kq_1^2}{r^2} \\\\F_1 = \frac{9\times 10^9 \times (1.6\times 10^{-19})^2}{r^2} \\\\F_1 = \frac{2.3 \times 10^{-28}}{r^2}[/tex]

The force on the electron due to uranium ion at distance less than 61.10 nm.

R = 61.10 nm - r

[tex]F_2 = \frac{9\times 10^9 \times (3.2 \times 10^{-19})^2}{(61.1 \times 10^{-9} \ - \ r)^2} \\\\\F_2 = \frac{9.216 \times 10^{-28}}{(61.1 \times 10^{-9} \ - \ r)^2}[/tex]

At equilibrium, the force between the electron and ions will be equal.

[tex]\frac{9.216 \times 10^{-28}}{(61.1 \times 10^{-9} \ - \ r)^2}= \frac{2.3 \times 10^{-28}}{r^2}\\\\\frac{4}{(61.1 \times 10^{-9} \ - \ r)^2} = \frac{1}{r^2} \\\\4r^2 = (61.1 \times 10^{-9} \ - \ r)^2\\\\2^2r^2 = (61.1 \times 10^{-9} \ - \ r)^2\\\\2r = 61.1 \times 10^{-9} \ - \ r\\\\3r = 61.1 \times 10^{-9} \\\\r = \frac{61.1 \times 10^{-9}}{3} \\\\r = 2.04 \times 10^{-8} \ m[/tex]

The magnitude of the force on the electron from the uranium ion is calculated as follows;

[tex]F = \frac{kq_1^2}{r^2} \\\\F = \frac{9\times 10^9 \times 1.6\times 10^{-19}}{(2.04 \times 10^{-8})^2} \\\\F= 3.46 \times 10^6 \ N[/tex]

Learn more here:https://brainly.com/question/16796365

Please help! THIS IS A EASY ONE, HOPEFULLY.

Answers

Answer:

megaliter > kiloliter > liter >centiliter >mililiter > deciliter > nanoliter

Explanation:

plz mark brainlest

.................,,,,,,,,,,,

Answers

Answer:

B

Explanation:

Motion is movement, the teacher's movement is motion

How does the angle of launch affect the kinetic energy of a rubber band?​

Answers

Answer:

The angle of launch of the rubber band affects the initial velocity. The more the rubber band is stretched the more force it applies to return to equilibrium and the more kinetic energy that results in.

A 2.0-kilogram ball traveling north at 4.0 meters per second collides head on with a 1.0-kilogram ball traveling south at 8.0 meters per second. What is the magnitude of the total momentum of the two balls after collision?

Answers

Answer:

We know the momentum after the collision MUST be equal to the momentum BEFORE the collision.  

Momentum is a VECTOR quantity having both magnitude and direction.  The first ball has momentum P =m*v = 2*4 = 8 at 90degrees.  The second ball has momentum P = 1*8 = 8 at -90 or 270 degrees.  They sum to zero when you perform vector addition.

Explanation:

A car turns a certain curve of radius 24.98 m with constant linear speed of
15.67 m/s. If the centripetal force experienced by that car is 34.652 kN, what is the
mass of the car?

Answers

Answer:

3525.19 kg

Explanation:

The computation of the mass of the car is shown below:

As we know that

Fc = m × V^2 ÷ R

m = Fc × R ÷ V^2

Provided that:

Fc = 34.652 kN = 34652 N

R = Radius = 24.98 m

V = speed = 15.67 m/s

So,

m = 34652 × 24.98 ÷ 15.67^2

 = 3525.19 kg

According to the article, what was the effect of elevation on the experimental group?

Answers

Answer:

Due to the effect of elevation on the experimental group the participants decided to try to help the research assistant who was having opening one of her files to finish the study. Schnall, Roper, and Fessler were able to conclude that happiness associated with a feeling of elevation can lead to more altruism or helping behaviors.

Explanation:

100 percent on edge

The effect of elevation on the experimental group was they show more urge of being altruistic and feeling happiness and satisfaction associated with elevation.

Altruism:

It is a practice in which a person help others without any selfishness, the person just want to help.

Witnessing someone's altruistic behavior, make other to feel good and a urge of being Altruistic, this is known as elevation.

After watching elevating Oprah video, the test group help the research assistant who was having trouble in opening file.

Therefore, the effect of elevation on the experimental group was they show more urge of being altruistic and feeling happiness and satisfaction associated with elevation.

To know more about Altruism:

https://brainly.com/question/25776081

The magnitude obtained when adding vector A (80 N at 20 deg) with vector B (40 N at

70 deg) is:

110.06 N

89.85 N

0 130.32 N

0 141.98 N

Answers

Answer:

110.06N

Explanation:

The magnitude of the force is known as the resultant.

R = √Fx²+Fy²

Fx = 80cos 20 + 40cos70

Fx = 80(0.9397)+40(0.3420)

Fx = 75.176 + 13.68

Fx = 88.856N

Fy = 80sin 20 + 40sin70

Fy = 80(0.3420)+40(0.9397)

Fy = 27.36 + 37.588

Fy = 64.948N

R = √88.586²+64.948²

R = √7,847.48+4,218.24

R = √12,065.72

R = 109.5

R = 110N

Hence the magnitude of the forces is 110N

How much force will a 5 kg rock hit the Earth with if it falls
for 1 second?

Answers

Answer:

f

Explanation:

f

The Great Sandini is a 60 kg circus performer who is shot from a cannon (actually a spring gun). You don't find many men of his caliber, so you help him design a new gun. This new gun has a very large spring with a very small mass and a force constant of 1300 N/m that he will compress with a force of 6500 N. The inside of the gun barrel is coated with Teflon, so the average friction force will be only 50 N during the 5.0 mm he moves in the barrel.

Required:
At what speed will he emerge from the end of the barrel, 2.5 mabove his initial rest position?

Answers

Answer:

22m/s

Explanation:

Mass, m=60 kg

Force constant, k=1300N/m

Restoring force, Fx=6500 N

Average friction force, f=50 N

Length of barrel, l=5m

y=2.5 m

Initial velocity, u=0

[tex]F_x=kx[/tex]

Substitute the values

[tex]6500=1300x[/tex]

[tex]x=\frac{6500}{1300}=5[/tex]m

Work done due to friction force

[tex]W_f=fscos\theta[/tex]

We have [tex]\theta=180^{\circ}[/tex]

Substitute the values

[tex]W_f=50\times 5cos180^{\circ}[/tex]

[tex]W_f=-250J[/tex]

Initial kinetic energy, Ki=0

Initial gravitational energy, [tex]U_{grav,1}=0[/tex]\

Initial elastic potential energy

[tex]U_{el,1}=\frac{1}{2}kx^2=\frac{1}{2}(1300)(5^2)[/tex]

[tex]U_{el,1}=16250J[/tex]

Final elastic energy,[tex]U_{el,2}=0[/tex]

Final kinetic energy, [tex]K_f=\frac{1}{2}(60)v^2=30v^2[/tex]

Final gravitational energy, [tex]U_{grav,2}=mgh=60\times 9.8\times 2.5[/tex]

Final gravitational energy, [tex]U_{grav,2}=1470J[/tex]

Using work-energy theorem

[tex]K_i+U_{grav,1}+U_{el,1}+W_f=K_f+U_{grav,2}+U_{el,2}[/tex]

Substitute the values

[tex]0+0+16250-250=30v^2+1470+0[/tex]

[tex]16000-1470=30v^2[/tex]

[tex]14530=30v^2[/tex]

[tex]v^2=\frac{14530}{30}[/tex]

[tex]v=\sqrt{\frac{14530}{30}}[/tex]

[tex]v=22m/s[/tex]

The spring is unstretched at the position x = 0. under the action of a force p, the cart moves from the initial position x1 = -8 in. to the final position x2 = 5 in. determine (a) the work done on the cart by the spring and (b) the work done on the cart by its weight.

Answers

This question is incomplete, the missing diagram is uploaded along this Answer below.

Answer:

a) the work done on the cart by the spring is 4.875 lb-ft

b) the work done on the cart by its weight is - 3.935 lb-ft

Explanation:

Given the data in the question;

(a) determine the work done on the cart by the spring

we calculate the work done on the cart by the spring as follows;

[tex]W_{spring}[/tex] = 1/2×k( [tex]x^{2} _{1}[/tex] - [tex]x^{2} _{2}[/tex] )

where k is spring constant ( 3 lb/in )

we substitute  

[tex]W_{spring}[/tex] = 1/2 × 3( (-8)² - (5)² )      

[tex]W_{spring}[/tex] = 1/2 × 3( 64 - 25 )

[tex]W_{spring}[/tex] = 1/2 × 3( 39 )

[tex]W_{spring}[/tex] = 58.5 lb-in

we convert to pound force-foot

[tex]W_{spring}[/tex] = 58.5 × 0.0833333 lb-ft

[tex]W_{spring}[/tex] = 4.875 lb-ft

Therefore, the work done on the cart by the spring is 4.875 lb-ft

b) the work done on the cart by its weight

work done by its weight;

[tex]W_{gravity}[/tex] = -mgsin∅( x₂ - x₁ )        

we substitute in of values from the image below;

[tex]W_{gravity}[/tex] = -14 × sin(15°)( 5 - (-8) )  

[tex]W_{gravity}[/tex] = -14 × 0.2588 × 13

[tex]W_{gravity}[/tex] = -47.1  lb-in

we convert to pound force-foot

[tex]W_{gravity}[/tex] = -47.1 × 0.0833333 lb-ft

[tex]W_{gravity}[/tex] = - 3.935 lb-ft

Therefore, the work done on the cart by its weight is - 3.935 lb-ft

a) the work done on the cart by the spring is 4.875 lb-ft.

b) the work done on the cart by its weight is - 3.935 lb-ft.

Calculation of the work done:

a. The work done on the cart by the spring is

= 1/2 × 3( (-8)² - (5)² )      

= 1/2 × 3( 64 - 25 )

= 1/2 × 3( 39 )

= 58.5 lb-in

Now we have to convert to pound force-foot

So,

= 58.5 × 0.0833333 lb-ft

= 4.875 lb-ft

b) Now

work done by its weight;

= -mgsin∅( x₂ - x₁ )        

So,

= -14 × sin(15°)( 5 - (-8) )  

= -14 × 0.2588 × 13

= -47.1  lb-in

Now we convert to pound force-foot

= -47.1 × 0.0833333 lb-ft

= - 3.935 lb-ft

Learn more about spring here: https://brainly.com/question/16060099

An object is placed in material a at point P, as shown in the diagram. The light is refracted when it strikes the interface with material b. When viewed from material b, at which point will the image appear?

Answers

B because the areas of f can be 7 or higher
Other Questions
How did Albrecht Drer use the element of value in his painting, "View of Trento"? How does it affect the way you see the painting? I will mark Brainlist so try your best Many of the DD-amino acids found in nature have been discovered in bacterially produced peptides that have antibiotic properties.a. Trueb. False Pick a small appliance in your home that you use regularly and look up on the internet how much power it operates at. Using the household voltage of 120 V, calculate the current flowing in this device. How much resistance would a resistor operating at that power and voltage have? If electricity costs $0.12/kWh, estimate how much you spend each month using this appliance. When you make your video, be sure to show the appliance you are talking about. ABC Co. has an average collection period of 45 days and an operating cycle of 130 days. It has a policy of keeping at least $10 on hand as a minimum cash balance, and has a beginning cash balance for the first quarter of $10. Beginning receivables for the quarter amount to $35. Sales for the first and second quarters are expected to be $110 and $125, respectively, while purchases amount to 80% of the next quarter's forecast sales. The accounts payable period is 90 days. What is the ending cash balance for the first quarter When a gas is heated, the gas particles have more kinetic energy. Complete the following sentence: This means they move about more and have more ______ with the sides of the container. This increases the pressure. make up 2 words using the word phobia with their meanings During exercise, the heart rate increases to: A. Supply more oxygen and glucose to the respiring cells. B. Supply more lactic acid to the muscles for anaerobic respiration. C. None of these D. Keep up with the pulse rate. Equation with Rational Numbers 8.4x-5=20.2 solves for x.. Construct the electron dot diagram for chlorine. You have completed this exercise. Step 1: Use the periodic table to find the chemical symbol for chlorine. The symbol for chlorine is: Civ Step 2: How many valence electrons does chlorine have? Step 3: Add dots one by one around the chemical symbol to represent valence electrons. Start at the top and move clockwise around the symbol. CT: Nice work! You have created the electron dot formula for chlorine. Intro ) Final Do you believe animals can adapt to environmental changes? Explain. what is CHA-CHA-CHA. In 3 paragraph minimum, how do we stop Bullying? Please help ASAP. Thanks so much. How to get the simple interest Which crop leaves soil with the lowest levels of nitrogen? So, I think the Knave of Spades is telling the truth because he wasn't accused of anything but im not sure. Brainliest goes to whoever answers the question properly :> When is the proper time to eat when you exercise? 39 - 15x if x = 2Evaluate HELP PLEASE!! Please help me with this! Will mark Brainlist!